You are on page 1of 73

Nurses2Canada

Welcome skannath

2) According to Piaget's theory of cognitive development, a 6-month-old infant should be


demonstrating:

Options:

A) Early traces of memory

B) Beginning sense of time

C) Repetitious reflex responses

D) Beginning of object permanence

Correct Answer is: D

Explanation : The concept of object permanence begins to develop around 6 months of age.

24) A child is diagnosed as having type I diabetes. The school nurse sets up a plan of care that
includes:

Options:

A) Limiting fluid intake during school hours


B) Asking the child each day what was eaten for breakfast

C) Noting the presence of diabetes in the file but treating the child like other children

D) Checking daily for injuries because of participation in the physical education program

Correct Answer is: C

Explanation : It is important to be aware of the disorder and have documentation, but it is more
important that the child be treated in the same way as other children.

25) When caring for the newborn of a mother with diabetes, the nursery nurse should be alert
for a sign of hypoglycemia, which is

Options:

A) Poor sucking reflex

B) Extreme restlessness

C) Excessive birth weight

D) Pallor of the skin and mucosa

Correct Answer is: A

Explanation : Feeding difficulties are due to hypoglycemic effects on the fetal CNS.

47) A parent of a five-year-old child who was recently diagnosed with type I diabetes asks a
nurse when the child can begin to self-administer insulin. The nurse would recommend that the
child begin this procedure at age.

Options:

A) Six

B) nine

C) twelve

D) fifteen
Correct Answer is: B

Explanation : From about nine years of age, children can be taught to administer their own
insulin. At this age they are able to understand the principles behind administration and
determine the dosage of insulin

50) Which of the following findings, if identified in a child by a school nurse, would be consistent
with physical abuse?

Options:

A) Changeable behavior and bruises

B) Tearfulness and insomnia

C) Decreased attention span and poor appetite

D) Nervousness and palpitations

Correct Answer is: A

Explanation : Unexplained bruises and behavioral extremes are symptoms of physical abuse

52) A newborn who is being cared for in an open warming unit has an axillary temperature of
96.2 F (35.7 C). It is essential that the nurse take which of the following actions?

Options:

A) Wrap the newborn in a blanket

B) Notify the parents of the findings

C) Increases the heat-control setting on the warming unit

D) Perform a heel stick to check the capilary blood glucose

Correct Answer is: C

Explanation : Increasing the temperature of the warming unit is the action of choice.
53) A nurse should recognize that an infant would receive which of the following vaccinations
during the neonatal period?

Options:

A) Haemophilus influenza B

B) Rubella

C) Hepatitis B

D) Varicella

Correct Answer is: C

Explanation : Hepatitis B vaccination is recommended for all infants. The first dose of the vaccine
may be given at birth or at one to two months of age in infants born to healthy women. Infants
born to an infected mother or a mother who is a chronic carrier should be given the vaccine as
well as hepatitis B immunoglobulin within 12 hours of birth. Parental consent should be
obtained before administering the vaccine

54) The nurse is reviewing the American Dietetic Association (ADA) diet with a 10-year-old child
who has diabetes mellitus. The child's selection of which of the following foods in exchange for a
serving of meat indicates that the child correctly understands the instructions?

Options:

A) Cheese omelet

B) Bacon

C) Chocolate milk

D) Baked beans

Correct Answer is: A

Explanation : The diabetic exchange list for meat includes the choice of cheese and eggs.
55) When a newborn is 12-hours-old, the nurse is to give him his first bath. The nurse should
initially obtain which of the following assessments?

Options:

A) Temperature

B) Weight loss since birth

C) Size of posterior fontanel

D) Passage of meconium

Correct Answer is: A

Explanation : The infant's temperature must be stable prior to bathing.

65) A nurse at outpatient clinic is returning phone calls that have been made to the clinic. Which
of the following calls should have the highest priority for medical intervention?

Options:

A) A home health patient reports, “I am starting to have breakdown of my heels.” B:

B) A patient that received an upper extremity cast yesterday reports, “I can’t feel my fingers in
my right hand today.”

C) A young female reports, ”I think I sprained my ankle about 2 weeks ago.”

D) A middle-aged patient reports, ”My knee is still hurting from the TKR.”

Correct Answer is: B

Explanation : The patient experiencing neurovascular changes should have the highest priority.
Pain following a TKR is normal, and breakdown over the heels is a gradual process. Moreover, a
subacute ankle sprain is almost never a medical emergency.

66) A nurse is covering a pediatric unit and is responsible for a 15 year-old male patient on the
floor. The mother of the child states, “I think my son is sexually interested in girls.” The most
appropriate course of action of the nurse is to respond by stating:
Options:

A) “I will talk to the doctor about it.”

B) “Has this been going on for a while?”

C) “How do you know this?”

D) “Teenagers often exhibit signs of sexual interest in females.”

Correct Answer is: D

Explanation : Adolescents exhibiting signs of sexual development and interest are normal.

68) Which of the following foods is NOT high in iron?

Options:

A) meat

B) black beans

C) seafood

D) green vegetables

Correct Answer is: B

Explanation : It is important to get plenty of iron in the diet

69) Which of the following is NOT a characteristic of vitamin C deficiency?

Options:

A) brittle bones

B) headache

C) loose teeth

D) tiny peripheral hemorrhages


Correct Answer is: B

Explanation : Brittle bones, tooth and gum problems, and small hemorrhages are all
characteristic of vitamin C deficiency

70) What is the single best way to inhibit the spread of microorganisms?

Options:

A) washing your hands

B) destroying old clothes

C) sanitizing all equipment

D) reusing latex gloves

Correct Answer is: A

Explanation : Hand should be washed before and after any contact with patients.

72) Which blood serum finding in a client with diabetic ketoacidosis alerts the nurse that
immediate action is required?

Options:

A) pH below 7.3

B) Potassium of 5.0

C) HCT of 60

D) Pa O2 of 79%

Correct Answer is: C

Explanation : HCT of 60. This high hematocrit is indicative of severe dehydration which requires
priority attention in diabetic ketoacidosis. Without sufficient hydration, all systems of the body
are at risk for hypoxia from a lack of or sluggish circulation. In the absence of insulin, which
facilitates the transport of glucose into the cell, the body breaks down fats and proteins to
supply energy ketones, a by-product of fat metabolism. These accumulate causing metabolic
acidosis (pH < 7.3), which would be the second concern for this client. The potassium and PaO2
levels are near normal.

73) The nurse is preparing a handout on infant feeding to be distributed to families visiting the
clinic. Which notation should be included in the teaching materials?

Options:

A) Solid foods are introduced one at a time beginning with cereal

B) Finely ground meat should be started early to provide iron

C) Egg white is added early to increase protein intake

D) Solid foods should be mixed with formula in a bottle

Correct Answer is: A

Explanation : Solid foods are introduced one at a time beginning with cereal. Solid foods should
be added one at a time between 4-6 months. If the infant is able to tolerate the food, another
may be added in a week. Iron fortified cereal is the recommended first food.

74) The nurse is performing a physical assessment on a toddler. Which of the following actions
should be the first?

Options:

A) Perform traumatic procedures

B) Use minimal physical contact

C) Proceed from head to toe

D) Explain the exam in detail

Correct Answer is: B

Explanation : Use minimal physical contact. The nurse should approach the toddler slowly and
use minimal physical contact initially so as to gain the toddler''s cooperation. Be flexible in the
sequence of the exam, and give only brief simple explanations just prior to the action.

80) A nurse is assigned to give a child a tepid tub bath to treat hyperthermia. Following the bath,
the nurse plans to:

Options:

A) Place the child in bed and cover the child with a blanket

B) Leave the child uncovered for 15 minutes

C) Assist the child to put on a cotton sleep shirt

D) Take the child’s axillary temperature in 2 hours

Correct Answer is: C

Explanation : Cotton is a lightweight material that will protect the child from becoming chilled
after the bath. Option 1 is incorrect because a blanket is heavy and may increase the child’s body
temperature and further increase metabolism. Option 2 is incorrect because the child should not
be left uncovered. Option 4 is incorrect because the child’s temperature should be reassessed in
one half hour after the bath.

81) A nurse instructs a hospitalized client in a low-fat diet. The client indicates understanding of
this diet by choosing which of the following from the dietary menu?

Options:

A) Liver, potato salad, sherbet

B) Shrimp and bacon salad

C) Turkey breast, boiled rice, and angel food cake

D) Lean hamburger steak, macaroni and cheese

Correct Answer is: C

Explanation : Major sources of fats include meats, salad dressings, eggs, butter, cheese, and
bacon. Options 1, 2, and 4 contain high-fat foods.
82) A nurse is teaching health education classes to a group of expectant parents, and the topic is
preventing mental retardation caused by congenital hypothyroidism. The nurse tell the parents
that the most effective means of preventing this disorder is by:

Options:

A) Adequate protein intake

B) Limiting alcohol consumption

C) Vitamin intake

D) Neonatal screening

Correct Answer is: D

Explanation : Congenital hyptothyroidism is the most common preventable cause of mental


retardation. Neonatal screening is the only means of early diagnosis and subsequent prevention
of mental retardation. Newborn infants are screened for congenital hypothyroidism before
discharge from the nursery and before 7 days of life. Treatment is begun immediately if
necessary. Adequate protein and vitamin intake will not specifically prevent this disorder. Alcohol
consumption during pregnancy needs to be restricted, not limited.

83) A nurse has reinforced discharge instructions to a parent of a child following heart surgery.
Which statement by the parent would indicate a need for further instructions?

Options:

A) “I should call the physician if my child develops faster or harder breathing than normal.”

B) “My child can return to school for full days in 2 weeks following discharge.”

C) “I should have my child avoid crowds and people for 1 after discharge.”

D) “I should allow my child to play inside but omit play outside at this time.”

Correct Answer is: B

Explanation : The child may return to school the third week after hospital discharge, but the child
should go to school for half days for the first week. Play should be omitted for several weeks,
allowing inside play as tolerated. The child should avoid crowds of people for 1 week after
discharge, including crowds at day care centers and churches. If any difficulty with breathing
occurs, the parent should notify the physician.

84) Angela, a Public Health Nurse, is asked to create teaching tool that focuses on diabetes
management for clients who often eat meals outside of the home in restaurants. Angela invites a
group of diabetic clients to her workshop and realizes that the majority of participants are visual
learners. Based on this assessment, Angela should use which of the following teaching
techniques or tools to best assist this group of learners?

Options:

A) Handouts.

B) Lectures.

C) Discussion groups.

D) Question and answer time.

Correct Answer is: A

Explanation : Visual learners retain a greater amount of information for a longer period if the
presenter of the information reinforces the content by providing handouts. Visual learners prefer
information that is presented and supported in a handout format.

85) Angela, a Public Health Nurse, is asked to create teaching tool that focuses on diabetes
management for clients who often eat meals outside of the home in restaurants. Angela is
concerned about a client’s ability to retain the information she is presenting. Which of the
following techniques would enhance the retention of the material in the presentation?

Options:

A) Including a bibliography.

B) Using a lecture format.

C) Speaking very softly.

D) Using repetition.
Correct Answer is: D

Explanation : Repetition is an effective means of reinforcing critical information and enhancing


content retention.

86) Angela, a Public Health Nurse, is asked to create teaching tool that focuses on diabetes
management for clients who often eat meals outside of the home in restaurants. Angela
understands that before clients can learn, they must believe that they need to learn the
information. This is an example of which learning principle?

Options:

A) Maturation.

B) Relevance.

C) Initiative.

D) Motivation.

Correct Answer is: B

Explanation : Clients are more receptive and ready to learn if they see that the information is
real and relevant to them.

87) A client with newly diagnosed type 1 diabetes mellitus has been seen for three consecutive
days in the emergency department with hyperglycemia. During the assessment, the client says
to the nurse, I’m sorry to keep bothering you every day, but I just can’t give myself those awful
shots.” The nurse makes which therapeutic response?

Options:

A) You must learn to give yourself the shots.

B) I couldnt give myself a short either.

C) I’m sorry you are having trouble with your injections. Has someone given you instructions on
them?

D) Let me see if the doctor can change your medication.


Correct Answer is: C

Explanation : It is important to determine and deal with a client’s underlying fear of self-
injection. The nurse should determine if a knowledge deficit exists. Demanding a behavior or
skill is inappropriate (optional 1). Positive reinforcement is necessary instead of focusing on
negative behaviors (option 2). The nurse should not offer a change in regimen that can’t be
accomplished (option 4).

88) A nurse requests that a client with diabetes mellitus ask his or her significant other (s) to
attend an educational conference on self-administration of insulin. The client questions why
significant others need to be included. The nurse’s best response would be:

Options:

A) Clients and families often work together to develop strategies for the management of
diabetes.

B) Family members can take you to the doctor.

C) Family members are at risk of developing diabetes.

D) Nurses need someone to call and check on a client’s progress.

Correct Answer is: A

Explanation : Families and/or significant others may be included in diabetes education to assist
with adjustment to the diabetic regimen. Although options 2 and 3 may be accurate, they are
not the most appropriate response. Options 4 devalues the client and disregards the tissue of
independence and promotes powerlessness.

89) A nurse is assessing a client with Addison’s disease for signs of hyperkalemia. The nurse
expects to note which of the following if hyperkalemia is present?

Options:

A) Polyuria

B) Dry mucous membranes

C) Cardiac dysrhythmias

D) Prolonged bleeding time


Correct Answer is: C

Explanation : The inadequate production of aldosterone in Addison’s disease causes inadequate


excretion of potassium and results in hyperkalemia. The clinical manifestations of hyperkalemia
are the result of altered nerve transmission. The most harmful consequence of hyperkalemia is
its effect on cardiac function. Options 1, 2, and 4 are not manifestations associated with
Addison’s disease or hyperkalemia.

90) A nurse is admitting a client to the hospital who recently had a bilateral adrenalectomy.
Which intervention is essential for the nurse to include in the client’s plan of care?

Options:

A) Prevent social isolation

B) Discuss changes in body image

C) Consider occupational therapy

D) Avoid stress-producting situations and procedures

Correct Answer is: D

Explanation : Adrenalectomy can lead to adrenal insufficiency. Adrenal hormones are essential in
maintaining homeostasis in response to stressors. Options 1, 2, and 3 are not essential
interventions specific to this client’s problem.

91) A client is experiencing diabetes insipidus secondary to cranial surgery. The nurse who is
caring for the client plans to implement which of these anticipated therapies?

Options:

A) Fluid restriction

B) Intravenous (IV) replacement of fluid losses

C) Increased sodium intake

D) Administering diuretics
Correct Answer is: B

Explanation : the client with diabetes insipidus excretes large amounts of extremely dilute urine.
This usually occurs as a result of decreased synthesis or release of antidiuretic hormone (ADH) in
conditions such as head injury, surgery near the hypothalamus, or increased intracranial
pressure. Corrective measures include alloing ample oral fluid intake, administering IV fluid as
needed to replace sensible and insensible losses, and administering vasopressin (Pitressin).
Sodium is not administered because the serum sodium level is usually high, as is the serum
osmolality. Option 4 is incorrect.

92) A nurse who is caring for a client with Graves’ disease notes a nursing diagnosis of
“Imbalanced Nutrition: less than body requirements related to the effects of the hypercatabolic
state.” In the care plan. Which of the following indicates a successful outcome for this diagnosis?

Options:

A) The client maintains his normal weight or gradually gains weight if it is below normal

B) The client demonstrates knowledge regarding the need to consume a diet high in fat and low
in protein.

C) The client verbalizes the need to avoid snacking between meals.

D) The client discusses the relationship between mealtime and the blood glucose level

Correct Answer is: A

Explanation : Graves’ disease causes a state of chronic nutritional and caloric deficiency caused
by the metabolic effects of excessive T3 and T4. Clinical manifestations are weight loss and
increased appetite. It is therefore a nutritional goal that the client will not lose additional weight
and will gradually return to ideal body weight if necessary. To accomplish this, the client must be
encouraged to eat frequent high-calorie, high-protein, and high-carbohydrate meals and snacks.

93) A client with diabetes mellitus is told that amputation of the leg is necessary to sustain life.
The client is very upset and states to the nurse: “This is all the doctor’s fault! I have done
everything that the doctor has asked me to do!” The nurse interprets the client’s statement as:

Options:

A) An expected coping mechanism


B) A need to notify the hospital lawyer

C) An expression of guilt on the part of the client

D) An ineffective coping mechanism

Correct Answer is: A

Explanation : The expression of anger is known to be a normal response to impending loss, and
the anger may be directed toward self, God or other spiritual being, or the caregivers. The nurse
needs to be aware of the effective and ineffective coping mechanisms that can occur in a client
when loss is anticipated. Notifying the hospital lawyer is inappropriate. Guilt may or may not be
a component of the client’s feelings, and the data in the question does not provide an indication
that guilt is present.

94) A client with type diabetes mellitus was recently hospitalized for hyperglycemic hyderomolar
nonketotic syndrome (HHNS). Upon discharge from the hospital. The client expresses concerns
about the recurrence of HHNS. The nurse makes which statement to the client?

Options:

A) Don’t worry, your family will help you.

B) I’m sure this won’t happen again

C) You have concerns about the treatment for you condition?

D) I think you might need to go to the nursing home

Correct Answer is: C

Explanation : The nurse should provide time and listen to the client’s concerns. In option 3, the
nurse is attempting to clarify the client’s feelings. Option 1 and 2 provide inappropriate false
hope. Additionally, the nurse does not tell the client not to worry. Option 4 is not an appropriate
nursing response, disregards the client’s concerns, and gives advice.

95) A 10-year-old child who is very active socially and is often away from the parents has been
diagnosed with type 1 diabetes mellitus. The nurse prepares to educate the family about the
disorder and plans to teach:
Options:

A) The child’s school teacher to monitor insulin requirements and administer the child’s insulin

B) The child to monitor insulin requirements and administer own insulin

C) The parents to always be available to monitor the child’s insulin requirements

D) All the friends and family involved with the child’s activities to monitor the child’s insulin
requirements.

Correct Answer is: B

Explanation : Most children nine years ole or older can understand the principles of monitoring
their own insulin requirements. They are usually responsible enough to determine the
appropriate intervention needed to maintain their health. The school teacher will not take
responsibility for health carte interventions. Parents, friends, and family can not always be
available.

96) A nurse is caring for a client who has returned to the physician’s office for follow-up after a
parathyroidectomy with autotransplantation of some parathyroid tissue into the calcium and
vitamin D supplements since discharge 2 weeks ago. Which statement by the client indicates an
understanding of the medical management following this type of surgical procedure?

Options:

A) Well, I guess the transplant isn’t working because my calcium levels are still low.

B) The thought of taking these pills for the rest of my life makes me shudder!

C) I can’t wait for the transplant to start working. I’m tired of taking all these pills!

D) Do you think I’ll always have to take these pills?

Correct Answer is: C

Explanation : The purpose of autotransplantation of some parathyroid tissue to regain function


of the parathyroid gland. Autotransplantation of parathyroid tissue takes some time to mature.
Oral calcium and vitamin D supplements must be taken to prevent hypoparathyroidism until the
transplant matures and becomes an active endocrine gland.
97) A 9-year-old child is newly diagnosed with type 1 diabetes mellitus. The nurse is planning for
home care with the child and his family and determines that an age-appropriate activity for this
child for health maintenance is:

Options:

A) Independently self-administering insulin

B) Making independent decisions regarding sliding-scale coverage of insulin

C) Having an adult assist in self-administration of insulin and glucose monitoring

D) Administering insulin drawn up by an adult.

Correct Answer is: A

Explanation : School-aged children have the cognitive and motor skills to independently
administer insulin with adult supervision. Developmentally, they do not yet have the maturity to
make situational decisions without adult validation. Options 3 and 4 suppress the maximum level
of independence appropriate to the level of this child.

98) A nurse assesses the client with a diagnosis of thyroid storm. Which classic signs and
symptoms associated with thyroid storm would indicate the need for immediate nursing
intervention?

Options:

A) Polyuria, nausea, and severe headaches

B) Fever, tachycardia, and systolic hypertension

C) Profuse diaphoresis, flushing, and constipation

D) Hypotension, translucent skin, and obesity

Correct Answer is: B

Explanation : The excessive amounts of thyroid hormone cause a rapid increase in the metabolic
rate, thereby causing the classic signs and symptoms of thyroid storm such as fever, tachycardia,
and hypertension. When these signs present themselves, the nurse must take quick action to
prevent deterioration of the client’s health because death can ensure. Priority interventions
include maintaining a patent airway and stabilizing the hemodynamic status. Options 1, 3, and 4
do not indicate the need for immediate nursing intervention.

99) A client is admitted to the hospital for a thyroidectomy. While preparing the client for
surgery, the nurse assesses the client for psychosocial problems that may cause preoperative
anxiety, knowing that a realistic source of anxiety is fear of:

Options:

A) Developing gynecomastia and hirsutism postoperatively

B) Sexual dysfunction and infertility

C) Imposed dietary restrictions post discharge

D) Changes in body image secondary to the location of the incision

Correct Answer is: D

Explanation : Because the incision is in the neck area, the client may be fearful of having a large
scar postoperatively. Having all or part of the thyroid gland removed will not cause the client to
experience gynecomastia or hirsutim. Sexual dysfunction and infertility could possibly occur if
the entire thyroid gland was removed and if the client was not placed on thyroid replacement
medications. The client will not have specific dietary restrictions post-discharge.

100) A nurse administers 30 units of NPH insulin at 7:00 AM to a client with a blood glucose level
of 200 mg/dL. The nurse monitors the client for a hypoglycemic reaction, knowing that NPH
insulin peaks in approximately how many hours following administration?

Options:

A) 2 hours

B) 3 to 4 hours

C) 6 to 14 hours

D) 16 to 24 hours

Correct Answer is: C

Explanation : NPH is an intermediate-acting insulin with an onset of action in 60 to 120 minutes,


a peak time in 6 to 14 hours, and a duration time of 16 to 24 hours.

101) A client diagnosed as having catatonic excitement has been pacing rapidly non-stop for
several hours and is not eating or drinking. The nurse recognizes that in this situation:

Options:

A) There is an urgent need for physical and medical control

B) There is an urgent need for restraint

C) There is a need to encourage verbalization of feelings

D) The client will soon become catatonic stuporous

Correct Answer is: A

Explanation : Catatonic excitement is manifested by a state of extreme psychomotor agitation.


Clients urgently require physical and medical control because they are often destructive and
violent to others, and their excitement can cause them to injure themselves or to collapse from
complete exhaustion. Options 2, 3, and 4 are incorrect.

102) A psychotic client is pacing agitated, and presenting with aggressive gestures. The client’s
speech pattern is rapid, and the client’s affect is belligerent. Based on this objective data, the
immediate priority of care is to:

Options:

A) Provide safety for the client and other clients on the until

B) Bring the client to a less stimulated area to calm down and gain control

C) Provide the clients on the unit with a sense of comfort and safety

D) Assist the staff in caring for the client in a controlled environment

Correct Answer is: A

Explanation : If a client is exhibiting signs that indicate loss of control, the nurse’s immediate
priority is to ensure safety for all clients. Option 1 is the only option that addresses the client and
other clients safety needs. Option 2 addresses the client’s needs. Option 3 addresses other
clients’ needs. Option 4 is not client centered.

103) A moderately depressed client who was admitted t the mental health unit 2 days ago
suddenly begins smiling and reporting that the crisis is over. The client says to the nurse, “Call
the doctor. I’m finally cured.” The nurse interprets this behavior as a cue to modify the treatment
pan by:

Options:

A) Allowing off-unit privileges pm

B) Suggesting a reduction of medication

C) Allowing increased “In-room activities

D) Increasing the level of suicide precautions

Correct Answer is: D

Explanation : A client who is moderately depressed and has only been hospitalized 2 days is
unlikely to have such a dramatic cure. When a mood suddenly lifts, it likely that the client may
have made the decision to harm hi or herself. Suicide precautions are necessary to keep the
client safe.

104) A nurse planning care for a suicidal client. The nurse implements additional precaution at
which of the following times?

Options:

A) During the day shift

B) On weekdays

C) Between 8 AM to 10 AM

D) During shift change

Correct Answer is: D

Explanation : At shift change, there is often less availability of staff. The psychiatric nurse and
staff should increase precautions for suicidal clients at that time. Weekends are also high-risk
times, not weekdays. The night shift also presents a high-risk time.

105) A nurse is planning care for a hallucinating and delusional client who has been rescued from
a suicide attempt. The nurse plans to:

Options:

A) Check the whereabouts of the client every 15 minutes

B) Initiate suicide precautions with 30-minute checks

C) Initiate one-to-one suicide precautions

D) Ask that the client report suicidal thoughts immediately

Correct Answer is: C

Explanation : One-to-one suicide precautions are required for the client rescued from a suicide
attempt. In this situation, additional key information is that the client is delusional and
hallucinating. Both of these factors increase the risk of unpredictable behavior, decreased
judgement, and the risk of suicide. Options 1, 2, and 4 do not provide the constant supervision
necessary for this client.

106) A client with a diagnosis of recurrent major depression who is exhibiting psychotic
behaviors is admitted to the psychiatric unit. In creating a safe environment for the client, the
nurse most importantly develops a plan of care that deals specifically with the client’s:

Options:

A) Disturbed thought processes

B) Imbalanced nutrition: less than body requirements

C) Bathing/hygiene self-care deficit

D) Deficient knowledge

Correct Answer is: A

Explanation : Major depression, recurrent, with psychotic behaviors alerts the nurse that in
addition to the criteria that designate the diagnosis of major depression, one must also deal with
a client’s psychosis. Psychosis is defined as a state in which a person’s mental capacity to
recognize reality and communicate and relate to others is impaired, thus interfering with the
person’s capacity to deal with life’s demands. Disturbed thought processes generally indicates a
state of increased anxiety in which hallucinations and delusions prevail. Although options 2 and
3 are important option 1 is specific to the client. Option 4 is not a priority at this time.

107) A hospitalized client with a diagnosis of anorexia nervosa and in a state of starvation is in a
two-bed hospital room. A newly admitted client will be assigned to this client’s room. Which
client would be inappropriate to assign to this two-bed room?

Options:

A) A client with pneumonia

B) A client with a fractured leg that is casted

C) A client who can care for self

D) A client who is scheduled for a diagnostic test

Correct Answer is: A

Explanation : The client in a state of starvation has a compromised immune system. Having a
roommate with pneumonia would place the client at risk for infection. Options 2, 3, and 4 are
appropriate roommates.

108) A nurse observes a client wringing her hands and looking frightened. The client reports to
the nurse that she “feels out of control.” Which approach by the nurse is most appropriate to
maintain a safe environment?

Options:

A) Administer the prescribed prn antianxiety medication immediately

B) Move the client to a quiet room and talk about her feelings

C) Isolate the client in a “time-out” room

D) Observe the client in an ongoing manner but do not intervene.


Correct Answer is: B

Explanation : The anxiety symptoms demonstrated by this client require some form of
intervention. Moving the client to a quiet room decreases environmental stimulus. Talking
provides the nurse an opportunity to assess the cause of the client’s feelings and to identify
appropriate interventions. Isolation is appropriate if a client is a danger to self or others.
Medication is used only when other noninvasive approaches have been unsuccessful.

109) A nurse is planning activities for a depressed client who was just admitted to the hospital.
The nurse plans to:

Options:

A) Provide an activity that is quiet and solitary in nature to avoid increased fatigue, such as
working on a puzzle or reading a book

B) Plan nothing until the client asks to participate in the milieu

C) Offer the client a menu of daily activities and insist that the client participate in all of them

D) Provide a structured daily program of activities and encourage the client to participate

Correct Answer is: D

Explanation : A depressed person is often withdrawn. Also, the person experiences difficulty
concentrating, loss of interest or pleasure, low energy and fatigue, and feelings of worthlessness
and poor self-esteem. The plan of care needs to provide stimulation in a structured
environment. Options 1 and 2 are restrictive and offer little or no structure and stimulation. The
nurse should not insist that a client participate in all activities.

110) A client is admitted to the psychiatric unit following a suicide attempt by hanging. The
nurse’s most important aspect of care is to maintain client safety, and the nurse plans to:

Options:

A) Assign a staff member to the client who will remain with the client

B) Place the client in a seclusion room where all potentially dangerous articles are removed

C) Remove the client’s clothing and place the client in a hospital gown

D) Request that a client’s peer remain with the client at all times
Correct Answer is: A

Explanation : Hanging is a serious suicide attempt. The plan for care must reflect the action that
will promote the client’s safety. Constant observation by a staff member is necessary. It is not a
peer’s responsibility to safeguard a client. Removing one’s clothing does not maximize all
possible safety strategies. Placing the client in seclusion further isolates the client.

111) A nurse receives a telephone call from a male client who states that he wants to kill himself
and has a loaded gun on the table. The best nursing intervention is to:

Options:

A) Insist that the client give you his name an address so that you can get the police there
immediately

B) Keep the client talking and allow the client to ventilate feelings

C) Use therapeutic communication techniques, especially the reflection of feelings

D) Keep the client talking and signal to another staff member to trace the call so that appropriate
help can be sent

Correct Answer is: D

Explanation : In a crisis, the nurse must take an authoritative, active role to promote the client’s
safety. A loaded gun in the home of the client who verbalizes he wants to kill himself is a crisis.
The client’s safety is of prime concern. Keeping the client on the phone and getting help to the
client is the best intervention. Insisting may anger the client and be might hang up. Option 2
lacks the authoritative action stance of securing the client’s safety. Using therapeutic
communication techniques is important, but overuse of reflection may sound uncaring or
superficial and is lacking direction and a solution to the immediate problem of the client’s safety.

112) A client is being discharged form the hospital and will receive oxygen therapy at home. The
nurse is teaching the client and family about oxygen safety measures. Which statement by the
client indicates the need for further teaching?

Options:

A) “I realize that I should check the oxygen level of the portable tank on a consistent basis.”
B) “It is all right to burn my scented candles as long as they are a few feet away from my oxygen
tank.”

C) “I will not sit in front of my wood burning fireplace with my oxygen on.

D) “I will call the physician if I experience any shortness of breath.”

Correct Answer is: B

Explanation : Oxygen is a highly combustible gas, although it will not spontaneously burn or
cause in explosion. It can easily cause a fire to ignite in a client’s room if it contacts a spark from
a cigarette, burning candle, or electrical equipment. Options 1, 3, and 4 are appropriate oxygen
safety measures.

113) A hospitalized client with a history of alcohol abuse says to the nurse, “I am leaving now. I
have to go. I don’t want any more treatment. I have things. That I have to do right away.” The
client has not been discharged. In fact, the client is scheduled for an important diagnostic test to
be performed in 1 hour. After discussing the client’s concerns with the client, the client dresses
and begins to walk out of the hospital room. The appropriate nursing action is to:

Options:

A) Restrain the client until the physician can be reached

B) Call security to block al exit areas

C) Tell the client that he cannot return to this hospital again if he leaves now

D) Call the nursing supervisor

Correct Answer is: D

Explanation : A nurse can be charged with false imprisonment if clients are made to wrongfully
believe that they cannot leave the hospital. Most health care facilities have documents for
clients to sign that relate to the clients’ responsibilities when they leave against medical advice.
(AMA). The client should be asked to sign this document before leaving. The nurse should
request that the client wait to speak to the physician before leaving, but if the client refuses to
do so, the nurse cannot hold the client against his or her will. Restraining the client and calling
security to block exits constitutes the imprisonment. Any client has a right to health car and
cannot be told otherwise.
114) Two nurses are in the cafeteria having lunch in a quiet, secluded area. A physical therapist
from the physical therapy department joins the nurses. During lunch, the nurses discusses a
client who was physically abused. After lunch, the physical therapist provides therapy as
prescribed to this physically abused client and asks the client questions about the physical abuse.
The client discovers that the nurses told the therapist about the abuse situation and is
emotionally harmed. The ramifications associated with the nurses’ discussion about the client
are associated with which of the following?

Options:

A) None, because the discussion took place in a quiet, secluded area

B) They can be charged with slander

C) They can be charged with libel.

D) None, because the physical therapist is involved in the client’s care

Correct Answer is: B

Explanation : Defamation occurs when information is communicated to a third party that causes
damage to someone else’s reputation either in writing (libel) or verbal (slander). Common
examples are discussing information about a client in public areas or speaking negatively about
coworkers. The situation identified in the question can cause emotional harm to the client, and
the nurses could be charged with slander. This situation also violates the client’s right to
confidentiality.

115) When planning the discharge of a client with chronic anxiety, the nurse develops goals to
promote a safe environment at home. The appropriate maintenance goal for the client should
focus on which of the following?

Options:

A) Maintaining continued contact with a crisis counselor

B) Identifying anxiety-producing situations

C) Ignoring feelings of anxiety

D) Eliminating all anxiety from daily situations


Correct Answer is: B

Explanation : Recognizing situations that produce anxiety allows the client to prepare to cope
with anxiety or avoid a specific stimulus. Counselors will not be available for all anxiety-
producing situations. Additionally, this option does not encourage the development of internal
strengths. Ignoring feelings will not resolve anxiety. It is impossible to eliminate all anxiety from
life.

116) A male suicidal client is being discharged home with his family. Which statement by a family
member might constitute criteria for delaying discharge?

Options:

A) The client’s wife asks, “Does he know that I’ve already moved out and filed for a divorce?”

B) The client’s son states, “One of his friends visited last week to tell us Dad’s union is out on
strike.”

C) The client’s daughter states, “I’ve decided to postpone my wedding until Dad’s feeling better.”

D) The client’s brother asks, “Will my brother be able to continue as executor of our parent’s
trust?

Correct Answer is: A

Explanation : Single, divorced, and widowed clients have suicide rates that are greater than
those who are married. While the situation of the strike is stressful, the client will probably
receive a portion of his wages and can derive hope and a sense of belonging from being a
member of the union. While the client might feel responsible for his daughter’s postponement
of the wedding, if presented as an action to include him, the client will feel loved and cared for.
While being suicidal may reduce the ability to concentrate, if the client perceives the
executorship positively, taking the role away reinforces the client’s low self-esteem and
selfworth. This statement by the client’s brother also indicates a need for the client’s brother to
be educated about depressive illness.

117) A client with a diagnosis of depression admits that one reason for her depression is that too
many demands drain her energy. The client also admits that one reason the situation is bad is
because the word “no” is not part of her vocabulary when it comes to the requests and needs of
others. After 7 days of hospitalization, another client asks for assistance in cleaning the unit
Immediately. The client with depression says. “No, I can’t help you now. I am enjoying watching
this movie.” The nurse interprets this response as:

Options:

A) A shirking of responsibility

B) Withdrawal from peers

C) Increased control over decisions

D) Decreased cooperation with others

Correct Answer is: C

Explanation : The client has been unable to refuse requests in the past. Saying “no” now
indicates that the client is trying to meet her own needs. “No” is being said now without guilt
and apology. During the treatment process, the client has learned how to meet her own needs,
and this can help to maintain health upon discharge. Options 1, 2, and 4 are incorrect
interpretations.

118) A client with depression who was admitted to the mental health unit approximately 7 days
ago is preparing for discharge to home, and the nurse is evaluating the client’s understanding of
depression and coping strategies learned during hospitalization. The nurse determines that
further teaching needs to occur if the client makes which of the following statements?

Options:

A) “This hospital experience and the therapy has been a positive experience in my life.”

B) “I know I must continue to take my medications just as prescribed.”

C) “I now know that I can’t be all things to all people.”

D) “I know that I probably wont’ have depression in the future.

Correct Answer is: D

Explanation : Depression may be a recurring illness for some people. The client needs to
understand the symptoms of depression and recognize when or if treatment needs to begin
again. The other statements indicate that the client has learned some coping skills, such as
setting limits, taking medications, and reframing an a potentially unpleasant experience into a
more positive one.
119) A nurse is developing a plan of care for an older client with dementia and formulates a
nursing diagnosis of self-care deficit. The nurse develops which realistic outcome for the client?

Options:

A) The client will be admitted to a nursing home to have activities of daily living (ADL) needs met.

B) The client will function at the highest level of independence possible.

C) The client will complete all ADLs, independently within a 1-to 1 ½- hour time frame.

D) The nursing staff will attend to all of the client’s ADL needs during the hospital stay.

Correct Answer is: B

Explanation : All clients, regardless of age, need to be encouraged to perform at the highest level
of independence possible. This contributes to the client’s sense of control and well-being.
Options 1 and 4 are not client-centered goals. A 1-to 1 ½-hour time frame may not be realistic
for an older client with dementia.

120) A client with a history of depression will be participating in cognitive therapy for health
maintenance. The client says to the nurse. “How does this treatment work?” The nurse makes
which statement to the client?

Options:

A) “This type of treatment helps you examine how your thoughts and feelings contribute to your
difficulties.”

B) “This type of treatment helps you examine how your past life has contributed to your
problems.”

C) “This type of treatment helps you confront your fears by gradually exposing you to them.”

D) “This type of treatment will help you relax and develop new coping skills.”

Correct Answer is: A

Explanation : Cognitive therapy is frequently used with clients who have depression. This type of
therapy is based on exploring the client’s subjective experience. It includes examining the client’s
thoughts and feelings about situations as well as how these thoughts and feelings contribute to
and perpetuate the client’s difficulties and mood. Options 2, 3, and 4 are not characteristic of
cognitive therapy.

121) A nurse is caring for a client who has bipolar disorder and is in a manic state. The nurse
determines that which menu choice would be best for this client?

Options:

A) Scrambled eggs, orange juice, coffee with cream and sugar

B) Cheeseburger, banana, milk

C) Beef stew, fruit salad, tea

D) Macaroni and cheese, apple, milk

Correct Answer is: B

Explanation : The client in a manic state often has inadequate food and fluid intake as a result of
physical agitation. Foods that the client can eat “on the run” are best because the client is too
active to sit at meals and use utensils. Additionally, clients in a manic state should not have
caffeine-containing products.

122) A female client is admitted to the inpatient mental health unit. When asked her name, she
responds, “I am Elizabeth, the Queen of England.” The nurse recognizes this response as a(n):

Options:

A) Visual illusion

B) Auditory hallucination

C) Grandiose delusion

D) Loose association

Correct Answer is: C

Explanation : A delusion is an important personal belief that is almost certainly not true and
resists modification. An illusion is a misperception or misinterpretation of externally real stimuli.
A hallucination is a false perception. Loose association is thinking characterized by speech in
which ideas that are unrelated shift from one subject to another.

623) A client with obsessive compulsive disorder spends many hours during the day and night
washing her hands. When initially planning for a safe environment, the nurse allows the client to
continue this behavior because it:

Options:

A) relieves the client’s anxiety

B) Decreases the chance of infection

C) Gives the client a feeling of self-control

D) Increases self-esteem

Correct Answer is: A

Explanation : The compulsive act provides immediate relief from anxiety and is used to cope
with stress, conflict, or pain. Although the client may feel the need to increase self-esteem, that
is not the primary goal of this behavior. Options 2 and 3 are also incorrect interpretations of the
client’s need to perform this behavior.

624) A nurse observes an anxious client blocking the hallway, walking three steps forward and
then two steps backward. Other clients are agitated trying to get past. The nurse intervenes by:

Options:

A) Standing alongside the client and saying, “You’re very anxious today.”

B) Stopping the behavior and saying, “You’re going to get exhausted.”

C) Taking the client to the TV lounge and saying, “Relax and watch television now.”

D) Walking alongside the client and saying, “You’re not going anywhere very fast doing this.”

Correct Answer is: A

Explanation : An important consideration in alleviating the anxiety is to assist the client to


recognize their behavior. Options 2 and 3 do not address the increased anxiety and the need to
control the underlying behavior, and may even escalate the behavior. Option 4 does not raise the
client to a functioning level.

125) A manic client is placed in a seclusion room after an outburst of violent behavior that
involved a physical assault on another client. As the client is secluded, the nurse:

Options:

A) Remains silent because verbal interaction would be too stimulating

B) Tells the client that she well be allowed to rejoin the others when she can behave

C) Asks the client if she understands why the seclusion is necessary

D) informs the client that she is being secluded to help regain self-control

Correct Answer is: D

Explanation : the client is removed to a nonstimulating environment as a result of behavior.


Options 1, 2, and 3 are nontehrapeutic actions. Additionally, option 2 implies punishment. It is
best to directly inform the client of the purpose of seclusion.

126) A male client diagnosed with catatonic stupor demonstrates sever withdrawal by lying on
the bed with the body pulled into a fetal position. The nurse plans to:

Options:

A) Leave the client alone and intermittently check on him

B) Take the client into the dayroom with other clients so they can help watch him

C) Sit beside the client in silence and occasionally ask open-ended questions

D) Ask direct questions to encourage talking

Correct Answer is: C

Explanation : Clients who are withdrawn may be immobile and mute, and require consistent,
repeated approaches, intervention includes establishment of interpersonal contact. The nurse
facilitates communication with the client by sitting in silence, asking open-ended questions, and
pausing to provide opportunities for the client to respond. The client is not left alone. Asking
direct questions to this client is not therapeutic.

127) A nurse is interviewing a client on admission to the mental health inpatient unit who was
involved in a fire two months ago. The client is complaining of insomnia, difficulty concentrating,
nervousness, hypervigilance, and is frequently thinking about fires. The nurse assesses these
symptoms to be indicative of:

Options:

A) Obsessive compulsive disorder (OCD)

B) Phobia

C) Post-traumatic stress disorder (PTSD)

D) Dissociative disorder

Correct Answer is: C

Explanation : PTSD is precipitated by events that are overwhelming unpredictable, and


sometimes life-threatening. Typical symptoms of PTSD include difficulty concentrating, sleep
disturbances, intrusive recollections of the traumatic event, hypervigilance, and anxiety.

128) A nurse obtains an electrocardiogram (ECG) rhythm strip on a client who is anxious about
the result. The ECG shows that the rate is 90 beats per minute. To relieve anxiety, the nurse tells
the client that:

Options:

A) The rate is normal

B) There is no need to worry

C) Medication specific to the problem will be prescribed

D) A slower hear rate is preferred

Correct Answer is: A

Explanation : A normal adult resting pulse rate ranges between 60 and 100 beats per minute;
therefore, the rate is normal. The nurse would not tell a client “not to worry.” Options 3 and 4
indicate that the ECG is abnormal.

129) A client who recently had a gastrostomy feeding tube inserted refuses to participate in the
plan of care, will not make eye contact, and does not speak to the family or visitors. The nurse
assesses that the client is using which type of coping mechanism?

Options:

A) Self-control

B) Problem solving

C) Accepting responsibility

D) Distancing

Correct Answer is: D

Explanation : Distancing us an unwillingness or inability to discuss events. Self-control is


demonstrated by stoicism and hiding feelings. Problem solving involves making plans and
verbalizing what will be done. Accepting responsibility places the responsibility for a situation on
one’s self.

130) A nurse is monitoring a client for signs of alcohol withdrawal. Which assessment data
indicates early signs of withdrawal?

Options:

A) Anxiety, tremor, irritability

B) Disorientation, and sleepiness

C) Dizziness, vomiting, headache

D) Clouding of consciousness, tiredness, fatigue

Correct Answer is: A

Explanation : The signs of alcohol withdrawal develop within a few hours after cessation or
reduction of alcohol and peak after 24 to 48 hours. Early signs include anxiety, anorexia,
insomnia, tremor, irritability, an elevation in pulse and blood pressure nausea, vomiting, and
poorly formed hallucinations or illusions.

131) A nurse is performing an assessment on a 16-year-old female client who has been
diagnosed with anorexia nervosa. Which statement by the client would the nurse identify as a
priority requiring further assessment?

Options:

A) “I exercise 3 to 4 hours every day to keep my slim figure.”

B) “My best friend was in the hospital with this disease a year ago.”

C) “I’ve been told that I am 10% below ideal body weight.”

D) “I check my weight every day without fail.”

Correct Answer is: A

Explanation : Exercising 3 to 4 hours every day is excessive physical activity and unrealistic for a
16-year-old. The nurse needs to further assess this statement immediately to find out why the
client feels the need to exercise this much to maintain her figure. Although it’s unfortunate that
her best friend had this disease, this is not considered a major threat to this client’s physical
wellbeing. A weight that exceeds 15% below the ideal weight is significant with anorexia
nervosa. It is not considered abnormal to check weight every day. Many clients with anorexia
nervosa check their weight close to 20 times a day.

132) A client is admitted to the mental health unit with a diagnosis formulated for the client is
Disturbed Thought Process, secondary to paranoia. In formulating the plan of care with the
health care team, the nurse includes instruction to the staff to:

Options:

A) Avoid laughing or whispering in from of the client

B) Increase socialization of the client with peers

C) Have the client sign a release of information to appropriate parties so that adequate data can
be obtained for assessment purposes

D) Begin to educate the client about social supports in the community


Correct Answer is: A

Explanation : A client experiencing paranoia is distrustful and suspicious of others. The health
care team needs to establish rapport and trust with the client would increase the client’s
paranoia. Options 2, 3 and 4 ask the client to trust on a multitude of levels. These options are
too intrusive for a client who is paranoid.

133) A nurse is assessing a client who has a nursing diagnosis of risk for self-directed violence.
The client says, “You won’t have to worry about me much longer.” The nurse interprets this
statement as:

Options:

A) An expression of hopelessness

B) An expression of depression

C) The intention for self-mutilation

D) The intention of suicide

Correct Answer is: D

Explanation : A client with a risk for self-directed violence who says that he or she will not be
around much longer is making an expression of a suicidal intent. Although hopelessness,
depression, and self-mutilation may relate to self-directed violence, the statement that he or she
will not be around is a direct comment about the act of suicide.

134) A nurse notes that an assigned client is lying tense is bed staring at the cardiac monitor. The
client states, “There sure are a lot of wires around there. I sure hope we don’t get hit by
lightning.” The appropriate nursing response is which of the following?

Options:

A) “Would you like a mild sedative to help you relax?”

B) “Oh, don’t worry, the weather is suppose to be sunny and clear today.”

C) “Yes, all those wires must be a little scary. Did someone explain what the cardiac monitor was
for?”

D) “Your family can stay tonight if they wish.”


Correct Answer is: C

Explanation : The nurse should initially validate the client’s concern and then assess the client’s
knowledge regarding the cardiac monitor. This gives the nurse an opportunity to provide client
education of if necessary. Options 1, 2, and 4 do not address the client’s concern. Additionally,
pharmacological intervention should be considered only if necessary.

135) A family member of a client with a brain tumor states that he is distraught and feeling guilty
for not encouraging the client to seek medical evaluation earlier. The nurse would incorporate
which of the following items in formulating a response to the family member’s statement?

Options:

A) It is true that brain tumors are easily recognizable.

B) The symptoms of a brain tumor may be easily attributed to another cause.

C) Brain tumors are never detected until very late in their course.

D) There are no symptoms of a brain tumor.

Correct Answer is: B

Explanation : Signs and symptoms of a brain tumor vary depending on location and may easily be
attributed to another cause. Symptoms include headache, vomiting, visual disturbances, and
change in intellectual abilities or personality. Seizures occur in some clients. These symptoms can
be easily attributed to other causes. The family requires support to assist them in the normal
grieving process. Options 1, 3, and 4 are inaccurate.

136) A charge nurse observes a nursing assistant talking in an unusually loud voice to a client
with delirium. The charge nurse takes which action?

Options:

A) Speaks to nursing assistant immediately while in the client’s room to solve the problem

B) Informs the client that everything is all right

C) Ascertains the client’s safety, calmly asks the nursing assistant to join the nurse outside the
room, and informs the nursing assistant that her voice was unusually loud
D) Explains to the nursing assistant that yelling in the client’s room is tolerated only if the client is
talking loudly

Correct Answer is: C

Explanation : The nurse must ascertain that the client is safe, and then discuss the matter with
the nursing assistant in an area away from the hearing of the client. If the client heard the
conversation, the client may become more confused or agitated. Options 1, 2, and 4 are
incorrect actions.

137) A charge nurse observes a nursing assistant talking in an unusually loud voice to a client
with delirium. The charge nurse takes which action?

Options:

A) Speaks to nursing assistant immediately while in the client’s room to solve the problem

B) Informs the client that everything is all right

C) Ascertains the client’s safety, calmly asks the nursing assistant to join the nurse outside the
room, and informs the nursing assistant that her voice was unusually loud

D) Explains to the nursing assistant that yelling in the client’s room is tolerated only if the client is
talking loudly.

Correct Answer is: C

Explanation : The nurse must ascertain that the client is safe, and then discuss the matter with
the nursing assistant in an area away from the hearing of the client. If the client heard the
conversation, the client may become more confused or agitated. Options 1, 2, and 4 are
incorrect actions.

138) A nurse is leading a crisis intervention group. The clients are high school students who have
experienced a recent death of a classmate. The classmate committed suicide at the school, and
the clients are experiencing disbelief. The clients reviewed the details about finding the
classmate dead in a bathroom. Initially, the nurse would:

Options:

A) Inquire how the clients recovered from death in the past


B) Reinforce the client’s sense of growth through this death experience

C) Reinforce the client’s ability to work through this death event

D) Inquire about the client’s perception of their classmate’s suicide problem

Correct Answer is: D

Explanation : It is essential to determine the client’s views. Inquiring about the client’s
perception of the suicide will identify specifically the appraisal of the suicide and the meaning of
the perception. Options 2 and 3 are similar in terms of attempts to foster client’s self-esteem.
Such an approach is premature at this point. Although option 1 is exploratory, it does not
address the “here and now” appraisal in terms of their classmate’s suicide. Although the nurse is
interested in how clients have coped in the past, this inquiry is not the most immediate
assessment.

139) A client recovering from a head injury becomes agitated at times. Which action will most
likely calm this client?

Options:

A) Turn on the television to a musical program

B) Talk to the client about the familiar objects, such as family pictures, that are kept in the
client’s room

C) Assign the client a new task to master

D) Make the client aware that the behavior is undesirable

Correct Answer is: B

Explanation : Decreasing environmental stimuli aids in reducing agitation for the head-injured
client. Option 1 increases stimuli. Option 3 does not simplify the environment because a new
task may be frustrating. In option 4, the nurse uses negative reinforcement to help the client
adjust. Providing familiar objects will decrease anxiety.

140) A nurse has been working with a victim of rape in an outpatient setting for the past 4
weeks. The nurse identifies which client goal as an unrealistic short-term one?
Options:

A) The client will resolve feelings of fear and anxiety related to the rape trauma.

B) The client will experience physical healing of the wounds that were incurred at the time of the
rape.

C) The client will verbalize feelings about the rape event.

D) The client will participate in the treatment plan by keeping appointments and following
through with treatment options

Correct Answer is: A

Explanation : Short-term goals will include the beginning stages of dealing with the rape trauma.
Clients will be expected initially to keep appointments, participate in care, begin to explore
feelings, and begin to heal physical wounds that were inflicted at the time of the rape.
Resolution of feeling of anxiety and fear is a long term goal.

141) A client says to the nurse. “I’m going to die and I wish my family would stop hoping for a
cure! I get so angry when they carry on like this! After all, I’m the one who’s dying.” The nurse
makes which therapeutic response to the client?

Options:

A) “You’re feeling angry that your family continues to hope for you to be cured?”

B) “I think we should talk more about your anger at your family.”

C) “Well, it sounds like you’re being pretty pessimistic. After all, years ago people died of
pneumonia.”

D) “Have you shared your feeling with your family?”

Correct Answer is: A

Explanation : Reflection is the therapeutic communication technique that redirects the client’s
feelings back in order to validate what the client is saying. Option 1 uses the therapeutic
technique of reflection. In option 2, the nurse attempts to use focusing, but the attempt to
discuss central issues seems premature. In potion 3, the nurse makes a judgment and is
nontherapeutic in the one-to-one relationship. In option 4, the nurse is attempting to assess the
client’s ability to openly discuss feelings with family members. Although this is an appropriate
assessment for this client, the timing is somewhat premature and closes off facilitation of the
client’s feelings.

142) A nurse is caring for an older adult client who says, “I don’t want to talk with you. You’re
only a nurse. I’ll wait for my doctor.” The nurse makes which appropriate response to the client?

Options:

A) “I understand. I’ll leave you now and call your physician.”

B) “I’m assigned to work with you. Your doctor placed you in my hands.”

C) “You would prefer speak with your doctor?”

D) “I’m angry with the way you’re dismissed me. I am your nurse, not your servant.”

Correct Answer is: C

Explanation : In the correct option, the nurse uses reflection to redirect the client’s feelings back
for validation and focuses on the client’s desire to talk with the physician. Options 2 and 4 are
nontehrapeutic responses. Option 1 reinforces acceptance for the client to continue this
behavior.

143) A client says to the nurse, “I don’t do anything right. I’m such a loser.” The nurse makes
which therapeutic statement to the client?

Options:

A) “You do things right all the time.”

B) “Everything will get better.”

C) “You don’t do anything right?”

D) “You are not a loser, you are sick.”

Correct Answer is: C

Explanation : Option 3 provides the client the opportunity to verbalize. With this statement, the
nurse can learn more about what the client really means by the statement. Option 1,2, and 4 are
closed statements and do not encourage the client to explore further.
144) A client who is experiencing suicidal thoughts greets the nurse with the following
statement, “It just doesn’t seem worth it anymore. Why not just end it all?” The nurse would
further assess the client by making which of the following responses?

Options:

A) “I’m sure your family is worried about you.”

B) “I know you have had a stressful night.”

C) “Did you sleep at al last night?”

D) “Tell me what you mean by that?”

Correct Answer is: D

Explanation : Option 4 allows the client the opportunity to tell the nurse more about what his or
her current thoughts are. Option 1 is false reassurance and may block communication. While
option 2 is offering empathy to the client, it does not further assess. Option 3 changes the
subject and may block communication.

145) A client angrily tells a nurse that the doctor purposefully provided wrong information.
Which of the following responses would hinder therapeutic communication?

Options:

A) “I’m certain the doctor would not lie to you.”

B) “Can you describe the information that you are referring to?”

C) “I’m not sure what information you are referring to.”

D) “Do you think it would be helpful to talk to your doctor about this?”

Correct Answer is: A

Explanation : Option 1 hinders communication by disagreeing with the client. This technique
could make the client defensive and block further communication. Options 2 and 3 attempt to
clarify what the client is referring to. Option 4 attempts to explore if the client is comfortable
talking to the doctor about this issue and encourages direct confrontation.
146) A client with a diagnosis of major depression says to the nurse. “I should have died. I’ve
always been a failure.” The nurse makes which therapeutic response to the nurse?

Options:

A) “I see a lot of positive things in you.”

B) “Feeling like a failure is part of your illness.”

C) “You’ve been feeling like a failure for some time now?”

D) “You still have a great deal to live for.”

Correct Answer is: C

Explanation : Responding to the feelings expressed by a client is an effective therapeutic


communication technique. The correct option is an example of the use of restating. Options 1, 2,
and 4 block communication because they minimize the client’s experience and do not facilitate
exploration of the client’s expressed feelings.

147) A nurse is caring for a client who is diagnosed as having schizophrenia. The client is unable
to speak, although there is no known pathological dysfunction of the organs of communication.
The nurse documents that the client is experiencing:

Options:

A) Pressured speech

B) Verbigeration

C) Poverty of speech

D) Mutism

Correct Answer is: D

Explanation : Mutism is the absence of verbal speech. The client does not communicate verbally,
despite an intact physical structural ability to speak. Pressured speech refers to rapidity of
speech reflecting the client’s racing thoughts. Verbigeration is the purposeless repetition of
words or phrases. Poverty of speech means diminished amounts of speech or monotonic replies.
148) A client tells the nurse, “I am a spy for the FBI. I am an eye, an eye in the sky.” The nurse
recognizes that this is an example of:

Options:

A) Loosened associations

B) Echolalia

C) Clang associations

D) Word salad

Correct Answer is: C

Explanation : Repetition of words or phrases that are similar in sound and in no other (rhyming)
is one altered thought and language pattern seen in schizophrenia. Clang associations often take
the form of rhyming. Loosened associations occur when the individual speaks with frequent
changes of subject, and the content is only obliquely related. Echolalia is the involuntary parrot
like repetition of words spoken by others. Word salad is the use of words with no apparent
meaning attached to them or to their relationship to one another.

149) In performing a lethality assessment with a suicidal client, the nurse most appropriately
asks the client:

Options:

A) “Do you have any thoughts of ending it all?”

B) “Do you have any thoughts of killing yourself?”

C) “Do you wish your life was over?”

D) “Do you have a death wish?”

Correct Answer is: B

Explanation : A lethality assessment requires direct communication between the client and the
nurse concerning the client’s intent. It is important to provide a question that is directly related
to lethality. Euphemisms should be avoided.
150) A client tells the nurse, “My doctor says I can have the surgery and go home the same day
but I’m afraid. My husband’s dead and my son is 3,000 miles away. I’m alone and what happens
if something goes wrong? I’m not supposed to be up walking unless absolutely necessary.” The
nurse makes which therapeutic response to the client?

Options:

A) “I know, I know. They say, Managed Care is no Care! Have you got an alarm system so if you
fall, it will alert someone to come? If worse comes to worse, call me and I’ll come immediately.”

B) “Don’t worry. This procedure is done all the time without any problems. You’ll be fine!”

C) “Your concern is well voiced. I advise you to call your son and insist he come home
immediately! You can’t be too careful.”

D) “You seem very concerned about going home without help. Have you discussed your
concerns with your doctor?

Correct Answer is: D

Explanation : The client has verbalized concerns. In option 4, the nurse uses reflection to direct
the client’s feelings and concerns. In option 1, the nurse is ventilating the nurse’s own anger,
frustration, and powerlessness. In addition, the nurse is trying to problem solve for the client but
is overly controlling and takes the decision making out of the client’s hands. In option 2, the
nurse provides false reassurance and then minimizes the client’s concerns. In option 3, the nurse
is projecting the client’s own fears, and the problem solving suggested by the nurse will increase
fear and anxiety in the client.

151) During the nursing assessment, the client says, “My doctor just told me that my cancer has
spread and that I have less than 6 months to live.” Which of the following nursing responses
would be therapeutic?

Options:

A) “I know it seems desperate, but there have been a lot of breakthroughs. Something might
come along in a month or so to change your status drastically.”

B) “I hope you’ll focus on the fact that your doctor says you have 6 months to live and that you’ll
think of how you’d like to live.”

C) “I am sorry. There are no easy answer in times like this, are there?”
D) “I am sorry. Would you lie to discuss this with me some more?”

Correct Answer is: D

Explanation : The client has received very distressing news. The client is mot likely still in the
stage of shock and denial. In the correct option, the nurse invites the client to ventilate. Option 1
provides a social communication and false hope. Option 2 is patronizing and stereotypical.
Option 3 is social and expresses the nurse’s feelings rather than the client’s feelings.

152) A client with acute renal failure is having trouble remembering information and instructions
as a result of altered laboratory values. The nurse avoids doing which of the following when
communicating with this client?

Options:

A) Giving simple, clear directions

B) Explaining treatments using understandable language

C) Including the family in discussions related to care

D) Giving thorough, complete explanation of treatment options

Correct Answer is: D

Explanation : The client with acute renal failure may have difficulty remembering information
and instructions because of anxiety and altered laboratory values. Communications should be
clear, simple, and understandable. The family is included whenever possible. It is the physician/s
responsibility to explain treatment options.

153) An acutely psychotic client displays increased psychomotor activity. The nurse anticipates
that the physician will prescribe which of the following for the client?

Options:

A) Sertraline hydrochloride (Zoloft)

B) Haloperidol (Haldol)

C) Chloral Hydrate (Noctec)


D) Isocarboxazid (Marplan)

Correct Answer is: B

Explanation : Antipsychotics are used to treat acute and chronic psychosis, especially when the
client has increased psychomotor activity. A fast-acting, injectable agent would be the
medication of choice. Antidepressants (options 1 and 4) and hypnotics (option 3) are not
indicated for the presenting condition.

154) A female client who is in a manic state emerges from her room. She is topless and is making
sexual remarks and gestures toward staff and peers. The best initial nursing action is to:

Options:

A) Quietly approach the client, escort her to her room, and assist her in getting dressed

B) Approach the client in the hallway and insist that she go to her room

C) Confront the client on the inappropriateness of her behavior and offer her a time-out

D) Ask the other clients to ignore her behavior; eventually she will return to her room

Correct Answer is: A

Explanation : A person who is experiencing mania lacks insight and judgment, has poor impulse
control, and is highly excitable. The nurse must take control, and is highly excitable. The nurse
must take control without creating increased stress or anxiety to the client. “Insisting” the client
go to her room may meet with a great deal of resistance. Confronting the client and offering her
a consequence of “time-out” may be meaningless to her. Asking other clients to ignore her is
inappropriate. A quiet, firm approach while distracting the client (walking her to her room and
assisting her to get dressed) achieves the goal of having her dressed appropriately and
preserving her phychosocial integrity.

155) The spouse of a client who is dying says to the nurse, “I don’t think I can come anymore and
watch her die. It’s chewing me up too much!” The nurse makes which therapeutic response to
he spouse?

Options:

A) “I wish you’d focus on your wife’s pain rather than yours. I know it’s hard, but this isn’t about
what’s happening to you, you know.’

B) “I know it’s hard for you, but she would know if you’re not there and you’d feel guilty all the
rest of your days.”

C) It’s hard to watch someone you love die. You’ve been here with your wife every day. Are you
taking any time for yourself?”

D) “I think you’re making the right decision. Your wife knows you love her. You don’t have to
come. I’ll take care of her.”

Correct Answer is: C

Explanation : The most therapeutic response is the one that is empathic and reflects the nurse’s
understanding of the client’s (the husband) stress and emotional pain. In the correct option, the
nurse suggests that the client take time for himself. Option 1 is an example of a nontherapeutic,
judgmental attitude that places blame. Option 2 makes statements that the nurse cannot know
are true (the client may, in fact, not know if the husband visits) and predicts guilt feelings, which
is inappropriate. Option 4 fosters dependency and gives advice, which is nontherapeutic.

156) A client who is reported by the staff to be very demanding says to the nurse. “I can’t get any
help with my care! I call and call but the nurses never answer my light. Last night one of them
told me she had other patients besides me! I’m very sick, but the nurses don’t care!” the nurse
makes which therapeutic response to the client?

Options:

A) “I think you are being very impatient. The nurses work very hard and come as quickly as they
can.”

B) “I can hear your anger. That nurse had no right to speak to you that way. I will report her to
the Director. It won’t happen again.”

C) “It’s hard to be in bed and have to ask for help. You ring for a nurse should be fired.”

D) “You poor thing! I’m so sorry this happened to you. That nurse should be fired.”

Correct Answer is: C

Explanation : Empathy is a term that describes the nurse’s capacity to enter into the life of
another person and to perceive how the client is feeling and what meaning this has for the
client. In option 3, the nurse displays empathy and shares perceptions. Sharing perceptions asks
the client to validate the nurse’s understanding of what the client is feeling and thinking. It opens
the door for the client to share concerns, fears, and anxieties. In option 1, the nurse is assertive
and certainly defends the nursing staff as well. In option 2, the nurse expresses the client’s
frustration by labeling the client’s feelings an “angry” and disapproving of the nursing staff. This
is splitting and is nontherapeutic. Option 4 is a social response and is demanding to the client.

157) Family members who are awaiting the outcome of a suicide attempt are tearful. Which
statement by the nurse would be therapeutic to the family at this time?

Options:

A) “Don’t worry, you have nothing to feel guilty about.”

B) “Everything possible is being done.”

C) “Let me check to see how long it will be before you can see your loved one.”

D) “I can see you are worried.”

Correct Answer is: D

Explanation : options 1, 2, and 3 are communication blocks. Option 1 labels the family’s behavior
without their validation. Option 2 uses clichés and false reassurance. Option 3 focuses on an
important issue at an inappropriate time. Option 4 addresses the family’s feelings and displays
empathy.

158) A nurse is caring for a dying client who says, “What would you say if I asked you to be the
executor for my will?” Which nursing response would be therapeutic?

Options:

A) “Why, I’d be honored to be the executor of your will.”

B) “Is there any money in it? I adore money, but I am honest.”

C) “Your confidence in me is an honor, but I would like to understand more about your thinking.”

D) “I’d say, great! No worries. I’ll carry out your will just as your will just as you want me to.”
Correct Answer is: C

Explanation : In option 3, this nurse is seeking clarification and empathy. The client’s question
reflects the fact that the client has been thinking bout the will and how best to obtain an
executor. What is unknown is why the client is asking the nurse to be executor of the will and
other specific and important information. In addition, the nurse would want to investigate the
legal ramifications, which could arise if such a position was accepted. In option 1, the nurse
responds with a social communication with no assessment of the consequences, which is lacking
critical thinking and exploration of motivation or client needs. In option 2, the nurse uses
histrionic language and crass ideation. In option 4, the nurse provides false reassurance, which h
is nontherapeutic.

159) A client who is suffering from urticaria (hives)and pruritus says to the nurse, “What am I
going to do? I’m getting married next week and I’ll probably be covered in this rash and itching
like crazy.” The nurse makes which therapeutic response to the client?

Options:

A) “You’ve very troubled that this will extend into your wedding?”

B) “It’s probably just due to prewedding jitters.”

C) “The antihistamine will help a great deal, just you wait and see.”

D) “I hope your husband-to-be has a sense of humor.”

Correct Answer is: A

Explanation : the therapeutic communication technique that the nurse uses in option 1 is
reflection. In option 2, the nurse minimizes the client’s anxiety and fears. In option 3, the nurse
talks about antihistamines and asks the client to “wait and see.” This is nontherapeutic because
the nurse is making promises that may not be kept and because the response is close-ended and
shuts off the client’s expression of feelings. In option 4, the nurse uses humor inappropriately
and with insensivity.

160) A nurse is performing an assessment on a 14-year-old client. On assessment, the nurse


notes bruises and bleeding in the genital area, cigarette burns on the chest, rope burns on the
buttocks, and multiple old fractures. The child states, “I’m afraid to go home! My stepfather will
be angry with me for telling on him!” The nurse makes which therapeutic response to the child?

Options:
A) “I am sorry that this has happened to you, but you will be safe here. Your physician has
admitted you until further plans can be made.”

B) “You can’t go back there with that man. How do you think your mother will react?”

C) “You must know that your presence in the house will only tease your stepfather more.”

D) “Let’s keep this between you, me, and the physician until we can formulate further plans to
assist you.”

Correct Answer is: A

Explanation : A child who has been physically and sexually abused should be admitted to the
hospital. This will provide time for a more comprehensive evaluation while protecting the child
from further abusiveness. The correct option also provides an empathic statement that supports
the child to appropriately perceive self as the victim, while assuring the child of protection from
abuse. In option 2, the nurse does not respond with calm and reassuring communication style,
nor does the nurse maintain a professional attitude. Option 3, which holds an innuendo, appears
to accuse the victim of teasing the stepfather and is incorrect. It is also judgmental, controlling,
and demeaning. The nurse’s suggestion in option 4 is not only incorrect but is also passive in its
stance.

161) A nurse is assessing a client’s suicide potential. The nurse asks the client which most
important question?

Options:

A) “Why do you want to hurt yourself?”

B) “Can you describe how you are feeling right now?”

C) “Has anyone in your family committed suicide?”

D) “Do you have a plan to commit suicide?”

Correct Answer is: D

Explanation : When assessing for suicide risk, the nurse must evaluate if the client has suicide
plan. Clients who have a definitive plan pose a greater risk for suicide. Options 2 and 3 may also
be questions that the nurse would ask but are not the most important. The nurse avoids the use
of the word “why” when communicating with a client. The use of this word may place the client
on the defensive; additionally, the client may not even know the reason “why” he or she wants
to hurt self.

162) A nurse is caring for a client who is receiving electroconvulsive therapy (ECT) for a major
depressive disorder. Which assessment finding would the nurse identify as an unexpected side
effect of ECT requiring notifying the physician?

Options:

A) Memory loss

B) Disorientation

C) Confusion

D) Hypertension

Correct Answer is: D

Explanation : The major side effects of ECT are confusion, disorientation, and memory loss. A
change in blood pressure would not be an anticipated side effect and would be a cause for
concern. If hypertension occurred following ECT, the physician should be notified.

163) A nurse is performing a neurological assessment on a client with dementia and is assessing
the function of the frontal lobes of the brain. Assessment of which of the following items by the
nurse would yield the best information about this area of functioning?

Options:

A) Level of consciousness

B) Insight, judgment, and planning

C) Feelings or emotions

D) Eye movements

Correct Answer is: B

Explanation : Insight, judgment, and planning are part of the function of the frontal lobe. Level of
consciousness is controlled by the reticular activating system. Feelings and emotions are part of
the role of the limbic system. Eye movements are under the control of cranial nerves III, IV, and
VI.

165) A nurse plans to assess a client for the vegetative signs of depression. The nurse assesses
for these signs by determining the client’s:

Options:

A) Ability to think, concentrate, and make decisions

B) Appetite, weight, sleep patterns, and psychomotor activity

C) Level of self-esteem

D) Level of suicidal ideation

Correct Answer is: B

Explanation : The vegetative signs of depression are changes in physiological functioning during
depression. These include appetite, weight, sleep patterns, and psychomotor activity. Options 1,
3, and 4 represent psychological assessment categories.

166) An 18-year-old woman is admitted to a mental health unit with the diagnosis of anorexia
nervosa. The nurse plans care, knowing that health promotion should focus on:

Options:

A) Helping the client identify and examine dysfunctional thoughts and beliefs

B) Emphasizing social interaction with clients who are withdrawn

C) Providing a supportive environment

D) Examining intrapsychic conflicts and past issues

Correct Answer is: A

Explanation : Health promotion focuses on helping clients identify and examine dysfunctional
thoughts as well as identify and examine value and beliefs that maintain these thoughts.
Providing a supportive environment is important but is not as primary as option 1 for this client.
Emphasizing social interaction is not appropriate at this time. Examining intrapsychic conflicts
and past issues is not directly related to the client’s problem.

167) A nurse is preparing discharge plans for a hospitalized client who attempted suicide. The
nurse includes which of the following in the plan?

Options:

A) Weekly follow-up appointments

B) Contracts and immediately available crisis resources

C) Encouraging family and friends to always be present

D) Providing phone numbers for the hospital and physician

Correct Answer is: B

Explanation : Crisis times may occur between appointments. Contracts facilitate clients feeling a
responsibility for keeping a promise. This gives the client control. Family and friends cannot
always be present. Providing phone numbers will not ensure available and immediate crisis
intervention.

168) A nursing student is asked to conduct a clinical conference regarding autism. The student
plans to include in the discussion that the primary characteristic associated with autism is:

Options:

A) The consistent imitation of others actions

B) Normal social play

C) Lack of social interaction and awareness

D) Normal verbal but abnormal nonverbal communication

Correct Answer is: C

Explanation : Autism is a severe developmental disorder that begins in infancy or toddlerhood. A


primary characteristic is lack of social interaction and awareness. Social behaviors in autism
include lack of or abnormal imitations of others’ actions and the lack of or abnormal social play.
Additional characteristics include lack of or impaired verbal communication and marked
abnormal nonverbal communication.

169) A nurse is caring for a client in seclusion. The nurse determines that the client is safe to
come out of seclusion when the nurse hears the client say which of the following?

Options:

A) “I am no longer a threat to myself or others.”

B) “I need to use the restroom right away.”

C) “I’d like to go back to my room and be alone for a while.”

D) “I can’t breathe in here. The walls are closing in on me.”

Correct Answer is: A

Explanation : Option 1 indicates that the client may be safely removed from seclusion. The client
in seclusion must be assessed at regular intervals (usually every 15 to 30 minutes) for physical
needs, safety, and comfort. Option 2 indicates a physical need that could be met with a urinal,
bedpan, or commode. It does not indicate that the client has calmed down enough to leave the
seclusion room. Option 3 could be attempt to manipulate the nurse. It gives no indication that
the client will control him or herself when alone in the room. Option 4 could be handled by
supportive communication or a prn medication, if indicated. It does not necessitate
discontinuing seclusion.

170) An older male client who is a victim of elder abuse and his family have been seen in the
counseling center weekly for the past month. Which statement, if may by the abusive family
member, would indicate that he or she has learned more positive coping skills?

Options:

A) “I will be more careful to make sure that my father’s needs are 100 percent met.”

B) “I am so sorry and embarrassed that the abusive event occurred. It won’t happen again.”

C) “I feel better equipped to care for my father now that I know where to turn if I need
assistance.”

D) “Now that my father is moving into may home, I will have to stop drinking alcohol.”
Correct Answer is: C

Explanation : Elder abuse is sometimes the result of family members who are being expected to
care for their aging parents. This care can cause the family to become overextended, frustrated,
or financially depleted. Knowing where to turn in the community for assistance in caring for an
aging family member can bring much-needed relief. Using these alternatives is a positive coping
skill for many families. Options 1, 2, and 4 are statements of good faith or promises, which may
or may not be kept in the future.

171) A nurse is caring for a term infant who is 24 hours old who had a confirmed episode of
hypoglycemia at 1 hour of age. Which observation by the nurse would indicate the need for
further evaluation?

Options:

A) Blood glucose level of 40 mg/dL before the last feeding

B) High-pitched cry, eating 10 to 15 mL of formula per feeding

C) Weight loss of 4 ounces and dry, peeling skin

D) Breast-feeding for 20 minutes or greater, strong sucking

Correct Answer is: B

Explanation : At 24 hours of age, a term infant should be take able to consume at least one
ounce of formula per feeding. A high-pitched cry is indicative of neurological involvement. Blood
glucose levels are acceptable at 40 mg/dL in the first few days of life. Weight loss over the first
few days of life and dry, peeling skin are normal finding for term infants. Breast-feeding for 20
minutes with a strong suck is an excellent finding. Hypoglycemia causes central nervous system
symptoms (high-pitched cry) and also is exhibited by lack of strength in eating enough for
growth.

172) A client being discharged from the mental health unit has a history of anxiety and
command hallucinations to harm self or others. The nurse teaches the client about interventions
for hallucinations and anxiety. The nurse determines that the client understands these measures
when the client says:

Options:

A) “If I take my medication, I won’t be anxious.”


B) “I can call my clinical specialist when I’m hallucinating so that I can talk about my feelings and
plans and not hurt anyone.”

C) “I can go to group and talk about my feelings.”

D) “If I get enough sleep and eat well, I won’t get anxious and hear things.”

Correct Answer is: B

Explanation : There may be an increased risk for impulsive and/or aggressive behavior if a client
is receiving command hallucinations to harm self or others. The client should be asked if he or
she has intentions to hurt self or others. Talking about auditory hallucinations can interfere with
subvocal muscular activity associated with a hallucination. Options 1, 3, and 4 are general
interventions but are not specific to anxiety and hallucinations.

173) A nurse is assisting in planning care for a suicidal client newly admitted to the mental health
unit. In order to provide a caring, therapeutic environment, which of the following is included in
the nursing care plan?

Options:

A) Placing the client in a private room to ensure privacy and confidentiality

B) Establishing a therapeutic relationship and conveying unconditional positive regard

C) Placing the client in charge of a meaningful unit activity, such as the morning chess
tournament

D) Maintaining a distance of 10 inches at all times to ensure the client that control will be
provided.

Correct Answer is: B

Explanation : The establishment of a therapeutic relationship with the suicidal client increases
feelings of acceptance. While the suicidal behavior and thinking of the client is unacceptable, the
use of unconditional positive regard acknowledges the client in a human-to-human context and
increases the client’s sense of self-worth. The client would not be placed in a private room
because this is an unsafe action and may intensify the client’s feelings of worthlessness. Placing
the client in charge of the morning chess game is a premature intervention that can overwhelm
and cause the client to fail. This can reinforce the client’s feelings of worthlessness. Distances of
18 inches or less between two individuals constitutes intimate space. Invasion of this pace may
be misinterpreted by the client and increase the client’s tension and feelings of helplessness.

174) A nurse is engaged in preparing a client for electroconvulsive therapy (ECT). Following a
through discussion with the client and family, the client signs the informed consent. Upon
departure from the session, a family member states, “I don’t know . . . I don’t think that this ECT
will be helpful if it makes people’s memories worse.” The nurse would then:

Options:

A) Involve the family member in a dialogue to ascertain how the family member arrived at this
conclusive statement

B) Inquire with other family members and the client if they thought the same way about ECT
making people worse

C) Immediately reassure the client that the decision to receive ECT will help and that memory
loss of confusion is minimal and temporary

D) Reinforce with the family member that depression causes more memory impairment than
ECT

Correct Answer is: A

Explanation : In option 1, the nurse is exploring for data to assist in clarifying information about
the procedure with the family. Option 2 may place family members. Option 3 would not
acknowledge the family member’s statement and concern. Option 4 addresses content
clarification but not the assessment process and is not the most therapeutic action.1

175) A nurse is assigned to care for a client diagnosed with catatonic stupor. When the nurse
enters the client’s room, the client is found lying on the bed with the body pulled into a fetal
position. The nurse should:

Options:

A) Leave the client alone and continue with providing care to other clients

B) Take the client into the dayroom to be with other clients

C) Sit beside the client in silence and occasionally ask open-ended questions

D) Ask the client direct questions to encourage talking


Correct Answer is: C

Explanation : Clients who are withdrawn may be immobile and mute, and require consistent,
repeated approaches. Intervention includes establishment of interpersonal contact.
Communication with withdrawn clients requires much patience from the nurse. The nurse
facilities communication with the client by sitting in silence, asking open-ended questions, and
pausing to provide opportunities for the client to respond. The client would not be left alone.
Asking direct questions to the client is not therapeutic. It is not appropriate at this time to place
the client in a public place such as a dayroom.

176) A nurse is assigned to care for a client diagnosed with catatonic stupor. When the nurse
enters the client’s room, the client is found lying on the bed with the body pulled into a fetal
position. The nurse should:

Options:

A) Leave the client alone and continue with providing care to other clients

B) Take the client into the dayroom to be with other clients

C) Sit beside the client in silence and occasionally ask open-ended questions

D) Ask the client direct questions to encourage talking

Correct Answer is: C

Explanation : Clients who are withdrawn may be immobile and mute, and require consistent,
repeated approaches. Intervention includes establishment of interpersonal contact.
Communication with withdrawn clients requires much patience from the nurse. The nurse
facilities communication with the client by sitting in silence, asking open-ended questions, and
pausing to provide opportunities for the client to respond. The client would not be left alone.
Asking direct questions to the client is not therapeutic. It is not appropriate at this time to place
the client in a public place such as a dayroom.

177) A nurse is caring for a hospitalized client with a diagnosis of depression who is silent and
not communicating. The nurse develops a plan of care and incorporates strategies for
communicating with the client. Which statement would be most appropriate for the nurse to
make when caring for the client?
Options:

A) “Can you tell me how you are feeling today?

B) “Do you feel like talking today?”

C) “You are wearing your new shoes.”

D) “Can you tell me how you slept last night.”

Correct Answer is: C

Explanation : When a depressed client is mute or silent, the nurse should use the
communication technique of making observations. A statement such as “You are wearing you
new shoes.” is an appropriate statement to make to the client. When the client is not ready to
talk, direct questions (options 1, 2, and 4) can raise the client’s anxiety level. Pointing to
commonalties in the environment draws the client into and reinforces reality.

178) A nurse is caring for a client with delirium who states, “Look at the spiders on the wall.” The
nurse makes which response to the client?

Options:

A) “I can see the spiders on the wall, but they are not going to hurt you.”

B) “Would you like me to kill the spiders for you?”

C) “I know you are frightened, but I do not see spiders on the wall.”

D) “You’re having a hallucination; there are no spiders in this room at all.”

Correct Answer is: C

Explanation : When hallucinations are present, the nurse should reinforce reality with the client.
In option 3, the nurse addresses the client’s feelings and reinforces reality. Options 1 and 2 do
not reinforce reality. Option 4 reinforces reality but does not address the client’s feelings.

179) A nurse is caring for a client at risk for suicide. Which client behavior is most indicative that
the client may be contemplating suicide?

Options:
A) The client tells the nurse that he plans to use his shoelaces to strangle himself.

B) The client cries for long periods of time

C) The client spends long periods of time alone.

D) The client reports sleep disturbances.

Correct Answer is: A

Explanation : If a client displays a suicidal ideation and is able to share a plan, the client should
be taken very seriously and suicide precautions should be implemented. Option 1 clearly states
such a plan. Options 2, 3 and 4 are indicative of depression but are not as definitive as option 1
in regard to suicide.

180) A registered nurse asks a nursing student about suicide and suicide intentions. The
registered nurse determines that the student understands the concepts associated with this
topic if the student makes which statement?

Options:

A) “Suicide runs in the family, so there is nothing that health care personnel can do about it.”

B) “Suicidal attempts are just attention seeking behaviors.”

C) “Many individuals who commit suicide have talked about their suicidal intentions to others.”

D) “Only psychotic individuals commit suicide.”

Correct Answer is: C

Explanation : Most people who do commit suicide have given definite clues or warning about
their intentions. Suicide is not an inherited condition. A suicide attempt is not an attention-
seeking behavior, and each act should be taken very seriously. The individual who is suicidal is
not necessarily psychotic. Options 1, 2, and 4 are considered myths regarding suicide.

181) During electroconvulsive therapy (ECT), the client receives oxygen by mask via positive
pressure ventilation. The nurse assisting with this procedure knows that positive pressure
ventilation is necessary because:
Options:

A) Grand mat seizure activity depresses respirations

B) Muscle relaxants given to prevent injury during seizure activity depress respirations

C) Anesthesia is administered during the procedure.

D) Decreased oxygen to the brain increases confusion and disorientation

Correct Answer is: B

Explanation : A short-acting skeletal muscle relaxant such as succinylcholine (Anectine) is


administered during this procedure to prevent injuries during the seizure. The client receives
positive pressure ventilation until the muscle relaxant is metabolized, usually within 2 to 3
minutes. Options 1, 3, and 4 do not address the issue of the question and the specific reason for
positive pressure ventilation.

182) A manic client is placed in a seclusion room after an outburst of violent behavior that
included a physical assault on another client. As the client is secluded, the nurse should:

Options:

A) Remain silent because verbal interaction would b e too stimulating

B) Tell the client that he will be allowed to rejoin the others when he can behave

C) Ask the client if he understands why the seclusion is necessary

D) Inform the client that he is being secluded to help regain control of self

Correct Answer is: D

Explanation : The client is removed to a nonstimulating environment as a result of behavior.


Options 1, 2, and 3 are nontherapeutic. In addition, option 2 implies punishment. It is best to
directly inform the client of the purpose of the seclusion.

183) When planning the discharge of a client with chronic anxiety, the nurse evaluates
achievement of the discharge maintenance goals. Which goal would most appropriately have
been included in the plan of care requiring evaluation?
Options:

A) The client maintains contact with a crisis counselor

B) The client identifies anxiety producing situations

C) The client ignores feelings of anxiety.

D) The client eliminates all anxiety from daily situations.

Correct Answer is: B

Explanation : Recognizing situations that produce anxiety allows the client to prepare to cope
with anxiety or avoid specific stimulus. Counselors will not be available for all anxiety-producing
situations, and this option does not encourage the development of internal strengths. Ignoring
feelings will not resolve anxiety. It is impossible to eliminate all anxiety from life.

184) A client has received electroconvulsive therapy (ECT). In the post-treatment area and upon
the client’s awakening. The nurse will perform which intervention first?

Options:

A) Orient the client and monitor the client’s vital signs

B) Offer the client frequent reassurance and repeat orientation statements

C) Assist the client from the stretcher to a wheelchair

D) Check for a gag reflex and then encourage the client to eat breakfast and resume activity.

Correct Answer is: A

Explanation : The nurse would first monitor vital signs, orient the client, and review with the
client that he or she just received an ECT treatment. The post-treatment area should include
accessibility to the anesthesia staff, oxygen, suction, pulse oximeter, vital sign monitoring, and
emergency equipment. The nursing interventions outlined in options 2, 3, and 4 will follow
accordingly.

185) A client having a mild panic attack has the following arterial blood gas (ABG) results: pH
7.49, Pco2 31 mmHg, PAo2 97 mmHg. Hco3 22 mEq/L. The nurse reviews the results and
determines that the client has which acid-base disturbance?

Options:

A) Respiratory acidosis

B) Respiratory alkalosis

C) Metabolic acidosis

D) Metabolic alkalosis

Correct Answer is: B

Explanation : Acidosis is defined as a pH of less than 7.35, whereas alkalosis is defined as a pH of


greater than 7.45. Respiratory alkalosis is present when the PCo2 is less than 35, whereas
respiratory acidosis is present when the Pco2 is greater than 45. Metabolic acidosis is present
when the Hco-3 is greater than 27 mEq/L. This client’s ABGs are consistent with respiratory
alkalosis.

186) A physical assessment is performed on a suicidal client on admission to the inpatient unit.
The nurse understands that this is an important part of the admission process because it
provides the nurse with information regarding:

Options:

A) the presence of abnormalities

B) Evidence of physical self-harm

C) Existing medical problems

D) Baselina data

Correct Answer is: B

Explanation : the physical assessment of a suicidal client should be thorough and should focus on
the evidence of self-harm or the client’s formulation of a plan for the suicide attempt. Although
all of the options are correct, option 2 is most appropriate in the context of the suicidal client.
Clients with a history of self-harm are greater suicide risks.
187) A client being mechanically ventilated after experiencing a fat embolus is visibly anxious.
The nurse takes which appropriate action?

Options:

A) Encourages the client to sleep until arterial blood gas results improve

B) Asks a family member to stay with the client at al times

C) Asks the physician to obtain an order for an antianxiety medication

D) Remains with the client and provides reassurance

Correct Answer is: D

Explanation : The nurse always speaks to the client calmly and provides reassurance to the
anxious client. Family members are also stressed. Therefore, it is not beneficial to ask the family
to take on the burden of remaining with the client at all times. Encouraging the client to sleep
will not assist in relieving the client’s anxiety. Antianxiety medications are used only if necessary
and if other interventions fail to relieve the client’s anxiety.

188) A female client arrives at the emergency room and states she was just raped. In preparing a
plan of care, the priority intervention in addition to medical attention should include:

Options:

A) Providing instructions for medical follow-up

B) Obtaining counseling for the victim

C) Providing anticipatory guidance for police investigations, medical questions, and court
proceedings

D) Exploring safety concerns by obtaining permission to notify significant others who can provide
shelter

Correct Answer is: D

Explanation : After the provision of medical treatment, the nurse’s next priority would be
obtaining support and planning for safety. Options 2 and 3 seek to meet the emotional needs
related to the rape and emotional readiness for the process of discovery and legal action. Option
1 is concerned with ensuring that the victim understands the importance of and commits to the
need for medical follow-up. From the options provided, this is not a priority intervention.

189) Jessica Smith is 20 years old and has been diagnosed with depression. She is admitted to
the mental health unit, where she is encouraged to participate in group sessions. 1. John
Stewart, RN is assigned to work with Jessica asks for John if he is married or has a girlfriend. John
responds by saying, “I am curious what made you ask this question; however, What is important
is how you are feeling today.” John’s respond would be considered which of the following?

Options:

A) A. Inappropriate because Jessica was just interested in John’s personal situation.

B) A. Inappropriate because John should have answered to establish a herapeutic relationship.

C) A. Appropriate because John is neither married nor has a girlfriend.

D) A. Appropriate because the focus of a therapeutic relationship is on the client.

Correct Answer is: D

Explanation : Every nurse has a responsibility to practice in a manner that is consistent with
providing safe, competent, and ethical care. If John had shared personal information with
Jessica, he would have crossed the boundary of a therapeutic relationship and changed the focus
of the discussion from a client focus to a social focus. It is very important in all areas of care,
particularly in the mental setting, that the relationship between the nurse and the patient has
very clear boundaries and a client focus.

190) Jessica Smith is 20 years old and has been diagnosed with depression. She is admitted to
the mental health unit, where she is encouraged to participate in group sessions. 1. During the
night shift, Jessica tells a nurse that she is going to kill herself, and she is placed on constant
observation. When she asks to use the toilet, the nurse follows her into the bathroom. Jessica
says, “I don’t need you to follow me into the bathroom. Give me some space.” Which of the
following statements by the nurse would be considered the most appropriate?

Options:

A) “You are right. I don’t need to come into the bathroom with you. I will wait outside the Door.”

B) “I must stay with you until we are sure you are not going to hurt yourself.”

C) “If you think you are going to be alright, I will check on you in minutes.”
D) “I can’t imagine there is anything dangerous in the bathroom, so go ahead, and I will wait for
you in the hallway.”

Correct Answer is: B

Explanation : Jessica is depressed and has expressed suicidal thoughts. She has been placed on
constant supervision as required by the unit policy. Staying with Jessica, even when she is in the
bathroom, demonstrates an understanding of constant observation. Staying with the patient also
demonstrates exercising professional judgment regarding the policy and the situation.

191) Jessica Smith is 20 years old and has been diagnosed with depression. She is admitted to
the mental health unit, where she is encouraged to participate in group sessions. 1. Jessica is
ready to be discharged and the nurse.” It would be really good for me if we could meet for coffee
if I am feeling depressed again.” Which of the following statements indicates that the nurse
understand the boundaries of the therapeutic relationship?

Options:

A) “That would be okay as long as we go to a public place. Where would you like to meet?”

B) “Before you leave the hospital, I will make sure you have the information about the crisis
center.”

C) “We could go to the gym together. Exercise can be very therapeutic for patients experiencing
depression.”

D) “I often meet with people after they are Discharged. Sometime it is difficult to deal with
situations after you leave the hospital.”

Correct Answer is: B

Explanation : The nurse realizes that in addition to crossing boundaries within the therapeutic
relationship if they meet for coffee, it would not be consistent with promoting health and
wellness. Providing the number for the crisis worker at the crisis centre is an example of
promoting a healthy strategy if Jessica believes she is becoming depressed again.

192) A client is admitted to the hospital with Cushing’s syndrome. The nurse reviews the results
of the client’s laboratory studies for which manifestation of this disorder?
Options:

A) Hypokalemia

B) Hyperglycemia

C) Low white blood cell (WBC) count

D) Decreased plasma cortisol levels

Correct Answer is: B

Explanation : The client with adrenocorticosteroid excess experiences hyperkalemia,


hyperglycemia, elevated WBC count, and elevated plasma cortisol and adrenocorticotropic
hormone (ACTH) levels. These abnormalities are caused by the effects of excess glucocorticoids
and mineralocorticoids on the body.

193) A nurse has an order to discontinue the nasogastric tube of an assigned client. After
explaining the procedure to the client, the nurse raises the bed to a semi-Flower’s position,
places a towel across the chest, clears the tube with normal saline, clamps the tube, and
removes the tube:

Options:

A) During inspiration

B) After the client takes a deep breath

C) As the client breathes out

D) After expiration, but therefore inspiration

Correct Answer is: B

Explanation : Just before removing the tube, the client is asked to take a deep breath and hold it.
This action is important because the airway is partially occluded during tube removal. Also,
breath-holding minimizes the risk of aspirating gastric contents if spilled from the tube during
removal. The nurse pulls the tube out steadily and smoothly while the client holds the breath
options 1, 3, and 4 are incorrect.
194) A nurse is performing a health history on a client with chronic calcifying pancreatitis. The
nurse expects to most likely note which of the following Abdominal pain relieved with food
when obtaining information regarding the client’s health history?

Options:

A) 1. Abdominal pain relieved with food or antacids

B) Exposure to occupational chemicals

C) Weight gain

D) Chronic use of alcohol

Correct Answer is: D

Explanation : Chronic use of alcohol is the most frequent cause of chronic calcifying pancreatitis.
Abstinence from alcohol is important to prevent the client from developing chronic pancreatitis.
Clients usually experience malabsorption with weight loss. Pain will not be relieved with food or
antacids. Chemicals exposure is associated with cancer of the pancreas.

195) A physician has inserted a nasointestinal tube into a client. Following insertion, the nurse
tells the client to lie in which position to help the tube advance into the duodenum, past the
pyloric sphincter?

Options:

A) Supine with the head of the bad flat

B) Supine with head elevated 30 degrees

C) On the right side

D) On the left side

Correct Answer is: C

Explanation : Following insertion of a nasoenteric tube, the client is instructed to lie on the right
side to aid in the passage of the tube from the stomach into the duodream past the pyloric
sphincter. Options 1, 2, and 4 are incorrect positions.
196) A nurse is assisting the physician with the insertion of a Miller-Abbott tube. The nurse
understands that the procedure puts the client at risk for aspiration and implements which
action to decrease this risk?

Options:

A) Inserts the tube with the balloon inflated

B) Instructs the client to cough when the tube reaches the nasal pharynx

C) Places the client in a high-Flower’s position

D) Instructs the client to perform a Valsalva maneuver if the impulse to gag and vomit occurs

Correct Answer is: C

Explanation : The Miller-Abbott tube is a nasoenteric tube that is used to decompress the
intestine, as in correcting a bowel obstruction. Initial insertion of the tube is a physician
responsibility. The tube is a inserted with the balloon deflated in a manner similar to the proper
procedure for inserting a nasogastric tube. The client is usually given water to drink to facilitate
passage of the tube through the nasopharynx and esophagus. A high-Flower’s position
decreases the risk of aspiration if vomiting occurs.

197) A client is admitted to the hospital for a bowel resection following a diagnosis of a bowel
tumor. During the admission assessment, the client tells the nurse that a living will was prepared
3 years ago. The client asks the nurse if this document is still effective. The nurse makes which
response to the client?

Options:

A) “Yes it is.”

B) “You will have to ask you lawyer.”

C) “It should be reviewed yearly with your physician.”

D) “I have no idea.”

Correct Answer is: C

Explanation : The client should discuss the living will with the physician, and it should be
reviewed annually to ensure that it contains the client’s present wishes and desires. Option 1 is
incorrect. Option 4 is not at all helpful to the client and is in fact a communication block.
Although a lawyer would nee to be consulted if the living will needed to be changed, the
accurate nursing response would be to inform the client that the living will should be reviewed
annually.

198) A client who is scheduled for gallbladder surgery is mentally impaired and is unable to
communicate. With regard to obtaining permission for the surgical procedure, which nursing
intervention would be appropriate?

Options:

A) Ensure that the family has signed the informed consent

B) Ensure that the client has signed the informed consent

C) Inform the family about the advanced directive process.

D) Inform the family about the process of living will

Correct Answer is: A

Explanation : A client must be alert, able to communicate, and competent to sign an informed
consent. If the client is unable to, them the family can sign the consent. A living will lists the
medical treatment a person chooses to omit or refuse if the person becomes unable to make
decisions and is terminally ill. Advanced directives are forms of communication in which persons
can give direction on how they would like to be treated when they cannot speak for themselves.

199) A client has an order to take magnesium citrate to prevent constipation following upper and
lower gastrointestinal (GI) barium studies. The nurse tells the client that this medication is best
taken:

Options:

A) Chilled with a full glass of water

B) At room temperature

C) With a tepid glass of water

D) With fruit juice only


Correct Answer is: A

Explanation : Magnesium citrate is available as an oral solution. It is commonly used as a laxative


following certain studies of the GI tract. It should be served chilled and taken with a full glass of
water. It should not be allowed to stand for prolonged periods. Allowing the medication to stand
would reduce the carbonation and make the solution even less palatable. Options 2, 3, and 4 are
incorrect.

200) A client has just undergone an upper gastrointestinal (GI) series. The nurse provides which
of the following upon the client’s return to the unit as an important part of routine post-
procedure care?

Options:

A) Decreased fluids

B) Bland diet

C) NPO status

D) Mild laxative

Correct Answer is: D

Explanation : Barium sulfate, which is used as a contrast material during an upper GI series, is
constipating. If it is not eliminated from the GI tract, it can cause obstruction. Therefore,
laxatives of cathartics are administered as part of routine post-procedure care. Increased (not
decreased) fluids are also helpful but do not act in the same way as a laxative to eliminate the
barium. Options 2 and 3 are not routine post-procedure measures.

You might also like